LSAT and Law School Admissions Forum

Get expert LSAT preparation and law school admissions advice from PowerScore Test Preparation.

 Administrator
PowerScore Staff
  • PowerScore Staff
  • Posts: 8919
  • Joined: Feb 02, 2011
|
#26977
Complete Question Explanation

StrengthenX. The correct answer choice is (E)

The conclusion is presented at the end of this stimulus, and seems fairly reasonable: there is reason to question the financial magazine’s survey comparing North Americans’ concerns about personal finances to their concerns about politics. This conclusion is based on two premises, namely that questions in the survey seem clearly biased (the wording of the questions would influence the answers of those surveyed), and that the group participating in the survey (people who subscribe to a financial magazine) is self-selecting and thus unrepresentative of all North Americans. Since this is a Strengthen-Except question, the four incorrect answer choices will support the conclusion that there is reason to be skeptical about this survey. 
 
Answer choice (A) strengthens the author’s conclusion by calling into question the financial magazine’s credibility, thus supporting the idea that its conclusions/findings are known to be suspect.

Answer choice (B) provides a similar, but broader, piece of information telling us that the conclusions of most magazine surveys turn out to be false, thereby supporting the author’s claim that we should be skeptical here.

Answer choice (C) strengthens the author’s assertion by providing conflicting evidence, in that other surveys indicate an opposite conclusion is true and hence give more reason to doubt the conclusion of the survey referenced in the stimulus.

Answer choice (D) touches on the author’s two principle criticisms—the biased questions and the self-selecting sample—by stating that there is reason to be skeptical about surveys that commit these errors.
 
Answer choice (E) is correct because it fails to strengthen the author’s conclusion. The fact that North Americans also find social issues important does not have any effect on the credibility of the survey or on the author’s conclusion. The stimulus discusses the relative importance of only two concerns—personal finances and politics—so this choice is completely irrelevant to the argument and is thus the correct answer.
 egarcia193
  • Posts: 41
  • Joined: Jun 25, 2017
|
#36787
Hello,

Why is that D is wrong ? it does not seem to be strengthing the argument at all but merely stating that one should be skeptical of surveys that are biased or unrepresentative but does not tell us that the survey is biased isn't this just irrelevant information that does nothing to strengthen the argument? what am I missing?

Thanks
 Luke Haqq
PowerScore Staff
  • PowerScore Staff
  • Posts: 722
  • Joined: Apr 26, 2012
|
#36800
Hi egarcia193,

Answer (D) states, "(D) There is reason to be skeptical about the results of surveys that are biased and unrepresentative." In other words:
  • B & U :arrow: S
We know from the language of the stimulus that the survey results are biased and unrepresentative. Using the above diagram, since we have B and W, we would have S (reason to be skeptical). And the author's overall conclusion in the stimulus is that we should be skeptical about particular survey results. If (D) were true, the way it strengthens is that it helps bridge a link between the unrepresentativeness/bias that exists and the stimulus's conclusion that one should thus be skeptical about such surveys.
 tanushreebansal
  • Posts: 21
  • Joined: Jun 26, 2017
|
#38221
Hi Powerscore. I got the right answer E, but I had a hard time eliminating A and B.

I was skeptical of A and B at first because I thought they were erroneous thinking. For A, isn't it a flaw to judge the credibility of a statement (or in this case, survey) based off of the credibility of its source? And for B- isn't it also a flaw to generalize- just because most magazine surveys have generally been disproved doesn't mean this one has to be?

I guess my question really is if a statement is not sound logic, can it still strengthen an argument?
 Adam Tyson
PowerScore Staff
  • PowerScore Staff
  • Posts: 5153
  • Joined: Apr 14, 2011
|
#38597
There is a fine line between drawing a conclusion based on answers like A and B and using those answers to weaken a claim, tanushreebansal. While it would be a logical flaw to say "the magazine's credibility has been called into question before, so this survey is incorrect", it is NOT a flaw to say "because this magazine's credibility has been previously called into question, I am going to be somewhat skeptical about these survey results". Same thing with answer B - that info doesn't disprove the conclusions drawn in the survey, but it does give us a reason to doubt, to be skeptical, strengthening the author's claim that we should indeed be skeptical. All you need to weaken an argument is just some element of doubt, and all you need to strengthen it is just a little nudge in the direction of help.

On a "Strengthen-Except" question, our threshold goes even lower than usual. Since we need four answers that strengthen and one that does not strengthen, any answer that helps even a little (here, that would be any answer that introduces even the slightest doubt in the validity of the survey) is a wrong answer. We aren't looking for the answer that does the most to support the author here, we are looking for the one that does not support the author at all. A and B don't help much, if we are being purely logical, but they do help a little. The correct answer is completely irrelevant, having no impact on the argument, and that's why it is the best choice on this Stregthen-Except question.

I hope that helps! Good job spotting the logical flaws in those answers. Now we just need to work on how those flaws apply in varying situations.

Get the most out of your LSAT Prep Plus subscription.

Analyze and track your performance with our Testing and Analytics Package.